OG- FLAW in reasoning

This topic has expert replies
Master | Next Rank: 500 Posts
Posts: 152
Joined: Mon Feb 20, 2012 12:56 pm
Location: New york
Thanked: 9 times

OG- FLAW in reasoning

by agarwalva » Tue May 22, 2012 5:53 pm
An eyeglass manufacturer tried to boost sales for the summer quarter by offering its distributors a special
discount if their orders for that quarter exceeded those for last year's summer quarter by at least 20 percent.
Many distributors qualified for this discount. Even with much merchandise discounted, sales increased enough
to produce a healthy gain in net profits. The manufacturer plans to repeat this success by offering the same sort
of discount for the fall quarter.
Which of the following, if true, most clearly points to a flaw in the manufacturer's plan to repeat the successful
performance of the summer quarter?
60
(A) In general, a distributor's orders for the summer quarter are no higher than those for the spring quarter.
(B) Along with offering special discounts to qualifying distributors, the manufacturer increased newspaper and
radio advertising in those distributors' sales areas.
(C) The distributors most likely to qualify for the manufacturer's special discount are those whose orders were
unusually low a year earlier.
(D) The distributors how qualified for the manufacturer's special discount were free to decide how much of that
discount to pass on to their own customers.
(E) The distributors' ordering more goods in the summer quarter left them overstocked for the fall quarter.


OA E
Explain why C is incorrect. If the distributors who can qualify for the new fall discount had a unusual low sales last year.. means they will have a better sales this year.. there is no need for the discount.. Even without the discount there is going to be increase in sales
Last edited by agarwalva on Tue May 22, 2012 7:11 pm, edited 1 time in total.

Master | Next Rank: 500 Posts
Posts: 341
Joined: Sun Mar 25, 2012 6:59 pm
Thanked: 17 times
Followed by:4 members
GMAT Score:720

by ice_rush » Tue May 22, 2012 6:46 pm
Hi Agarwalva,

where does choice (C) mentions distributors who qualify for a discount in Fall? the special discount was offered in summer. choice (C) is referring to distributors who'd qualify for the special discount in summer.


hope this helps!

User avatar
Legendary Member
Posts: 502
Joined: Tue Jun 03, 2008 11:36 pm
Thanked: 99 times
Followed by:21 members

by vk_vinayak » Thu May 24, 2012 2:59 am
If the distributors who can qualify for the new fall discount had a unusual low sales last year.. means they will have a better sales this year
Why do you assume this? They might have had low sales because of the location, customer service, type of selection offered, etc.

E is clearly the best choice.
- VK

I will (Learn. Recognize. Apply)

User avatar
GMAT Instructor
Posts: 15539
Joined: Tue May 25, 2010 12:04 pm
Location: New York, NY
Thanked: 13060 times
Followed by:1906 members
GMAT Score:790

by GMATGuruNY » Thu May 24, 2012 7:58 am
agarwalva wrote:An eyeglass manufacturer tried to boost sales for the summer quarter by offering its distributors a special
discount if their orders for that quarter exceeded those for last year's summer quarter by at least 20 percent.
Many distributors qualified for this discount. Even with much merchandise discounted, sales increased enough
to produce a healthy gain in net profits. The manufacturer plans to repeat this success by offering the same sort
of discount for the fall quarter.
Which of the following, if true, most clearly points to a flaw in the manufacturer's plan to repeat the successful
performance of the summer quarter?
60
(A) In general, a distributor's orders for the summer quarter are no higher than those for the spring quarter.
(B) Along with offering special discounts to qualifying distributors, the manufacturer increased newspaper and
radio advertising in those distributors' sales areas.
(C) The distributors most likely to qualify for the manufacturer's special discount are those whose orders were
unusually low a year earlier.
(D) The distributors how qualified for the manufacturer's special discount were free to decide how much of that
discount to pass on to their own customers.
(E) The distributors' ordering more goods in the summer quarter left them overstocked for the fall quarter.


OA E
Explain why C is incorrect. If the distributors who can qualify for the new fall discount had a unusual low sales last year.. means they will have a better sales this year.. there is no need for the discount.. Even without the discount there is going to be increase in sales
The plan is to OFFER A DISCOUNT.
The conclusion is that SALES WILL BE BOOSTED.
Whether there is a NEED for the discount is irrelevant.
The ONLY question is whether the plan will work: if the discount is offered, will sales be boosted?
The correct answer must explain why sales will NOT be boosted.

Answer choice E: The distributors' ordering more goods in the summer quarter left them overstocked for the fall quarter.
If the distributors are OVERSTOCKED -- if they have TOO MUCH merchandise -- then they are unlikely to order more, weakening the conclusion that sales will be boosted.

The correct answer is E.

Answer choice C: The distributors most likely to qualify for the manufacturer's special discount are those whose orders were unusually low a year earlier.
Thus, the discount could induce these distributors to order MORE merchandise than they ordered last year, STRENGTHENING the conclusion that sales will be boosted.
Eliminate C.
Private tutor exclusively for the GMAT and GRE, with over 20 years of experience.
Followed here and elsewhere by over 1900 test-takers.
I have worked with students based in the US, Australia, Taiwan, China, Tajikistan, Kuwait, Saudi Arabia -- a long list of countries.
My students have been admitted to HBS, CBS, Tuck, Yale, Stern, Fuqua -- a long list of top programs.

As a tutor, I don't simply teach you how I would approach problems.
I unlock the best way for YOU to solve problems.

For more information, please email me (Mitch Hunt) at [email protected].
Student Review #1
Student Review #2
Student Review #3

User avatar
Legendary Member
Posts: 934
Joined: Tue Nov 09, 2010 5:16 am
Location: AAMCHI MUMBAI LOCAL
Thanked: 63 times
Followed by:14 members

by [email protected] » Wed Jun 20, 2012 12:18 am
Yes, option C strengthens the argument, even if it strengthens by only a margin let us say 5% or 10% or more than that. But the point is that it strengthens the argument.

Also, option E completely weakens the argument, 100%, certainty. According to the plan, the discount offered is the most important motivation to boost the sales. And option E very clearly says that the discount will no longer will be demanded by those departments, then how are you suppose to claim that the sales will boost up.

Thus the OA is E.
IT IS TIME TO BEAT THE GMAT

LEARNING, APPLICATION AND TIMING IS THE FACT OF GMAT AND LIFE AS WELL... KEEP PLAYING!!!

Whenever you feel that my post really helped you to learn something new, please press on the 'THANK' button.

Junior | Next Rank: 30 Posts
Posts: 10
Joined: Tue Nov 05, 2013 6:31 am

by tusharkhatri123 » Mon Jan 19, 2015 2:28 pm
Although I got my answer correct, but I pre-assumed something different than given in the answer choice. Please check whether this my pre-assumption is also correct:

The consumers had a demand for eyeglasses such as goggles in summer, but that demand reduced in fall as that was not the season goggles are meant for.

Thanks and Regards
Tushar

User avatar
GMAT Instructor
Posts: 1035
Joined: Fri Dec 17, 2010 11:13 am
Location: Los Angeles, CA
Thanked: 474 times
Followed by:365 members

by VivianKerr » Tue Jan 20, 2015 11:44 pm
This is a great example of a "Flaw" question -- a question-type that is so often overlooked! "Flaw" questions present you with a flawed argument. You must interpret exactly how the argument is flawed. Here are some common Logical Flaw question-stems:

"¢ Which of the following best describes the error of reasoning contained in the argument above?
"¢ Which of the following best describes a weakness present in the argument above?
"¢ The argument is flawed because it fails to consider that
"¢ The reasoning in the argument above is flawed because it presumes without justification
"¢ The argument above is most vulnerable to criticism on the ground that
"¢ The author's conclusion is based on the error in the reasoning that

Some of the most ways that arguments can be flawed:

- Generalization
- Baseless Conclusion
- Weak Assumption

This fits in the second category, as explained. Flaw questions fall into one of those three categories 90% of the time, so ANTICIPATE the flaw before you dive into the answer choices!
Vivian Kerr
GMAT Rockstar, Tutor
https://www.GMATrockstar.com
https://www.yelp.com/biz/gmat-rockstar-los-angeles

Former Kaplan and Grockit instructor, freelance GMAT content creator, now offering affordable, effective, Skype-tutoring for the GMAT at $150/hr. Contact: [email protected]

Thank you for all the "thanks" and "follows"! :-)